ASAP ASAP!! HELP!! A supermarket needs to display 64 boxes of detergent.
The boxes need to be stacked with the same number of boxes in each row.
There should be at least 3 rows with at least 4 boxes in each row.
Choose the correct answers from the drop-down boxes to complete all the ways the boxes of detergent can be stacked.


Choose...
rows with 16 boxes in each row;

8 rows with
Choose...
boxes in each row;

16 rows with
Choose...
boxes in each row.

Answers

Answer 1

Answer:

Its the first one

Step-by-step explanation:

4x4=16 8x2=16 so its the first

Answer 2

These three lines should say:

4 rows with 16 boxes in each row

8 rows with 8 boxes in each row

16 rows with 4 boxes in each row


Related Questions

Determine the number of solutions you will have for the system 4y + x = 16 and y = 4 – x.

Answers

Equaling both equations, it is found that the correct option that states the number of solutions you will have for the system is:

D: one solution

The equations are:

[tex]4y + x = 16 \rightarrow x = 16 - 4y[/tex]

[tex]y = 4 - x \rightarrow x = 4 - y[/tex]

Equaling both equations for x, we have that:

[tex]16 - 4y = 4 - y[/tex]

[tex]3y = 12[/tex]

[tex]y = \frac{12}{3}[/tex]

[tex]y = 4[/tex]

[tex]x = 4 - y = 4 - y = 0[/tex]

Hence, the system has one solution, which is (0,4), and option d is correct.

You can learn more about the number of solutions a system of equations has  at https://brainly.com/question/25960609

13,22,31,...
Find the 31st term.
Find the 31st term.


Answers

Answer: 283

Step-by-step explanation:

To do this, it is helpful to get an equation you can use to solve any term.

This equation is:

[tex]13 + 9(n-1)[/tex]

So simply plug in 31 for n to get

[tex]13+9(31-1)[/tex]

[tex]=13+270[/tex]

[tex]=283[/tex]

Answer:

283

Step-by-step explanation:

9x+4=9(31)+4=283

13+9=22

22+9=31

9x

9(1)=9 9+4=13

9x+4

Exercise on Properties/Laws of Sets Simplify each of the following identities to its simplest form, using the laws of sets; (i) (An B) U (AU B')'
(ii) An (AUB)
(iii) (CUA N (BUA)​

Answers

bro wat like bro what like what what bro.



Two angles are supplementary. The smaller angle is 35 degrees less than the larger angle. What is the measurement of the larger angle?

Answers

Answer:

Step-by-step explanation:

In Exercises 12-20, TA=AB=BC and TD= DE = EF

Write an Equation that relates AD and BE.

Answers

Based on the midsegment theorem of a triangle, the equation that relates AD and BE is: AD = ½(BE)

Recall:

The midsegment theorem of a triangle states that length of midsegment = ½(the third side of the triangle).

Given the image attached below, where, TA=AB=BC and TD= DE = EF, considering ΔTBE:

AD = midsegment of ΔTBEBE = third side of ΔTBE

Therefore, based on the midsegment theorem of a triangle, the equation that relates AD and BE is: AD = ½(BE)

Learn more about the midsegment theorem of a triangle on:

https://brainly.com/question/7423948

three movie tickets cost 36$ at this same rate how much do 12 tickets cost

Answers

Answer:

$144

Step-by-step explanation:

Each ticket is $12.

Given
f(x) = 3x^2+ 2x – 1 and g(x) =4x- 5find (gof)(-2)

Answers

Answer:

27

Step-by-step explanation:

yeah-ya......... right?

Find the value of sin D rounded to the nearest hundredth, if necessary.

Answers

Answer:

0.22

Step-by-step explanation:

So in this problem, we have to find the hypotenuse first, since sine is opposite over hypotenuse.

We can do this using the pythagorean theorem (you can search it if you don't know). Or, we can just remember this as one of our pythagorean triplets (9,40,41). The last side will be 41.

We have to find the value of sin(D)

opposite/hypotenuse = 9/41 = 0.2195 which is about 0.22

Planes A and B are shown.
If a new line, p, is drawn parallel to line I, which
statement is true?
ZA
O Line p must be drawn in plane B.
O Line p must be perpendicular to line m.
O Line p must be drawn so that it can lie in the same
plane as line 1
O Line p must be drawn in the same plane as line n.
m

Answers

B. line p must be tge perpendicular to line M

help with this question, please!

Answers

Answer:

Y Intercept: 0, 2.5

X Intercept: 3.5, 0

Explanation:

An Intercept is where a line crosses an axis. A Y Intercept is for the Y axis and the X intercept is for the X axis.

Also when writing coordinates, the x coordinate comes before the y one.

Hope this helps :)

Find the length of the third side. If necessary, round to the nearest tenth.

Answers

Answer:

Step-by-step explanation:

a² + b² = c²

12² + b² = 15²

b² = 225 - 144

b² = 81

b = 9

of the negative number, and of the
16
6 The sum of a negative number,
negative number is – 13.2. What is the negative number?

Answers

Answer:

semoga jawabannya benar

Step-by-step explanation:

Korelasi Rank Spearman digunakan untuk mencari hubungan atau untuk menguji signifikansi hipotesis asosiatif bila masing-masing variabel yang dihubungkan berbentuk Ordinal.

Answer:

179.2

Step-by-step explanation:

166 = x + -13.2

166 - -13.2 = 166 + 13.2 = 179.2

Help help help help help m at h math math

Answers

Answer:X= 6X-14

Step-by-step explanation:

1. The total amount of what you eam during the pay period before taxes and deductions are taken out is known as what?
Wage
Net pay
Salary
Gross pay

Answers

Answer:

net pay if im not mistaking

Step-by-step explanation:

I WILL GIVE BRAINLEST AND 60 POINTS I KNOW ITS NOT ALOT
Melanie was training for a race. The line plot below shows the number of miles Melanie ran last week.




2. How many times did Melanie run ½ a mile?

Answers

Answer:

Melanie and 2 times ½ a mile

Hopes This helps

Write the first five terms of the geometric sequence whose first terns 6, and whose common ratio is 3.

Answers

The first five terms of the geometric sequence is 6, 18, 54, 162 and 486

Geometric progression is given by:

[tex]nth\ term=ar^{n-1}[/tex]

where a is the first term, r is the common ratio

Given that first term is 6, and common ratio is 3, Hence a = 6, r = 3

[tex]First\ term=6\\\\Second \ term=6(3)^{2-1}=18\\\\Third \ term=6(3)^{3-1}=54\\\\Fourth \ term=6(3)^{4-1}=162\\\\Fifth \ term=6(3)^{5-1}=486[/tex]

The first five terms of the geometric sequence is 6, 18, 54, 162 and 486

Find out more on geometric sequence at: https://brainly.com/question/24355584

Which of the segments below is a secant?
С
F
О.
В
Д.
А
u
А. ОВ
B. AB
ООО О
C. DE
D. AD

Answers

Answer:

it is d i believe

The secant of a circle is line segment AD. Therefore, option C is the correct answer.

What is secant of a circle?

A line that crosses a circle at two different locations is said to be a secant of the circle. The Roman verb secare, which meaning to cut, is the source of the word secant. It can also be thought of as the chord of a circle extended outside of the circle.

Given that, circle with center O.

In the given figure, FC is chord, OB is radius, AB is tangent, AD is secant and DE is tangent.

Therefore, option C is the correct answer.

Learn more about the secant of a circle here:

https://brainly.com/question/23026602.

#SPJ7

can you help me with this graph question ​

Answers

4,-5
explaination: sorry if not correct!

Answer:

D

Step-by-step explanation:

the Max Point is (2,6) so y=b

sorry if u dont get it

Problem Situation: Ben buys a table for $240. He also buys 6 chairs that all cost the same amount. His total cost is $720. What is the price of one chair? Use the drop-down menus to complete parts A and B below.

Answers

The price of one chair Ben bought is $80 and he bought 6 chairs in total.

What is a numerical expression?

A numerical expression is a mathematical statement written in the form of numbers and unknown variables. We can form numerical expressions from statements.

Given,  Ben buys a table for $240. He also buys 6 chairs that all cost the same amount.

Assuming each chair costs $x and his total cost is $720.

240 + 6x = 720.

6x = 720 - 240.

6x = 480.

x = 80

learn more about numerical expressions here :

https://brainly.com/question/29199574

#SPJ1

2
Which function has a greater rate of change?
Function 1
Function 2
4
3
2
The function whose input x and
output y are related by
1
X+
2
-*x+7
14-3-2-1 1 2 3 4
2
A) Function 1
B) Function 2
C) Neither, they are equal
D) Neither, one is nonlinear

Answers

a) function 1 has a greater rate of change

Henry jumped up 6 steps. Each step was 3/10 feet high. How high did he jump all together (in simplest form)

Answers

Answer:

1.8 or 14/5

Step-by-step explanation:

HENry MuST Be A KaNGaRoO

Answer:

Step-by-step explanation:

Multiply "3/10 ft high per step" by "6 steps."  This works out to:

18/10 ft was the highest that this young man jumped.

This reduces to 9/5 ft, or 1.8 ft

Put these fractions in order from least to greatest:

Answers

Answer:

1/4, 3/8, 2/3

Step-by-step explanation:

3/8 is more than 25% (1/4) but is less than 66% (2/3) making the order 1/4 3/8 2/3.

Answer:

The answer is the 3rd option. (1/4, 3/8, 2/3)

Step-by-step explanation:

All u have to do is find a common denominator for all 3 fractions.

The common denominator in this problem would be 24

So lets convert all the fractions with a denominator of 24.

1/4 = 6/24

3/8 = 9/24

2/3 = 16/24

Since we now converted the fractions with a common denominator, we can deduce the order from least to greatest.

6/24, 9/24, then 16/24

In other words, the answer is the 3rd option. (1/4, 2/8, and 2/4)

Hope this helps! :0

૮ ・ﻌ・ა

write an expression that shows 9 less than the product of 11 and 3.5

Answers

try this for answer (11×3.5)-9=x

will give brainly if you help me thx

Answers

Answer:

A

Step-by-step explanation:

The answer is A because is it asking for a 3 in the hundredths place of the difference. Difference means subtraction. And you can find the hundredth spot in the second place over from the decimal

(ex. 1.23, hundredth place is a 3)

So then you do the math and find the differences.

14.56-9.33=5.23, which has a 3 in the hundredth place.

If Y = 13 inches, Z = 19 inches, H = 7 inches, and W = 3 inches, what is the area of the object?
A.
304 square inches
B.
123.5 square inches
C.
147 square inches
D.
190 square inches

Answers

Answer:

Step-by-step explanation:

Ask Siri

A TON OF POINTS AND BRAINLIEST IF ANSWERED CORRECTLY! I RLY NEED HELP PLZZZ ANSWER

Use the function f(x) = 2x^2 − 3x − 5 to answer the questions.

Part A: Completely factor f(x). (2 points)

Part B: What are the x-intercepts of the graph of f(x)? Show your work. (2 points)

Part C: Describe the end behavior of the graph of f(x). Explain. (2 points)

Part D: What are the steps you would use to graph f(x)? Justify that you can use the answers obtained in Part B and Part C to draw the graph. (4 points)

Answers

Answer:

Step-by-step explanation:

For  

f(x) = 2x2-3x -5

For Part A

Set f(x) = 0

Factor the Quadratic

Set the factors = 0  

Solve each factor for x to find your x intercepts

For Part B

You need to use the coefficients in your Quadratic  

List them

a = 2 , b = -3 and c = -5

Since a is positive, your parabola opens up  

Find the x coordinate of the vertex it is -b/2a

One you get the x coordinate, plug it into to your Quadratic to get f(x) your y coordinate of the vertex

For Part C

You can plot the vertex, both x intercepts on a coordinate grid

PLEASE HELP ME




Question 7 of 10

Which of the following is equal to the fraction below? (5/9)⁸

A. 5⁸/9⁸

B. 5/9⁸

c. 5⁸/9

D. 8•(5/9)

SUBMIT

Answers

Answer:

A 5⁸/9⁸

Step-by-step explanation:

the bracket there shows that the power is for both the numerator and denominator so A is the correct answer

EC
What is the Arabic numeral for this Roman numeral?
VII
07
09
08
STUR

Answers

Answer:

07

Step-by-step explanation:

Answer:

7

Step-by-step explanation:

V = 5

I = 1

V + I + I = 5 + 1 + 1 = 7

Hope that helps

How do you use observed data to predict unobserved

Answers

You can determine that the test scores will drop with more hours of tv watched. Though as you can also notice there seem to be more outliers later in the data so you can determine that what is I observed will have more outliers than what is observed.

3x+5y−7x+6=12
Calculate.Solve for x.​

Answers

The value of x from the equation is x = (5y - 6)/4

Given the equation 3x+5y−7x+6=12

collect the like terms;

3x-7x + 5y + 6=12

-4x + 5y = 12 - 6

-4x + 5y = 6

Subtract 6 from both sides

-4x + 5y - 5y = 6 - 5y

-4x = 6 - 5y

4x = 5y - 6

x = (5y - 6)/4

Hnce the value of x from the equation is x = (5y - 6)/4

learn more on subject of formula here: https://brainly.com/question/657646

Other Questions
Are the pigs the best option as leaders write a paragraph explaining. [URGENT NEED ANSWER!!!!!]Mettez en ordre1.examen/ un/avons /eu/ de/nous/hier /franais. 2. ai / derrire / vu / larbre / quelquun / j CAN SOMEONE PLZ HELP ME? :( The gravitational force on Mars is about 38% of Earth. What would a cat weighing 15 pounds on Earth weigh on Mars? find the missing side of each triangle What type of plate boundary is in the image below?DivergentConvergentTranform How can tone help you discover a narrator's motivation? v = 128 32tv is the velocity in ft/sec. It is NOT the height ofthe ball.t is the number of seconds after the ball isthrown.With what initial velocity is the ball thrown?128 secondssWhat is the y-intercept of the equation?IS[ Select]What does the y-intercept mean?t[ Select ]What is the x-intercept of the equation?[ Select]What does the x-intercept mean?Select]710--80F help me with this please will mark brainliest if correct Using the number lines shown, what is the equivalent fraction to 3/3 If anyone can figure the answer, please tell me. Anything would be helpful :)I have to master 20 lessons for khan all 100% and it's very difficult for some of them. need help w these ! ++10 points Order the following elements according to the valence numbers from lowest to highest1. Boron2. Sodium3. Chlorine4. Beryllium Kindly Simplify this Question 1 L = 0.26 gal How many liters are in 13.39 gallons? What is the domain? (Type your answer with commas and no spaces) which algebraic expression has like terms? a. 9n^3-2n+3-4n^2b. 7n^3+3n-3-6n^2c. 7n^3+4n-3n^3-5n^3d. 6n^3-4n^3+6n-5n^3 What was included in the Virginia Plan?a. Congress could make laws that states couldn'tb. Had three branches of governmentc. Congress could remove laws that violated the constitutiond. All of the above Given sin0= 3/7, and tan0 A tree that stands 75 feet tall, casts ashadow of 30 feet at 11:00 am. What isthe ratio of the length of the shadow to theheight of the tree?